2019-2020/1ST/Derivation/Nombre_derive/5E_etude_cas.tex

57 lines
2.6 KiB
TeX

\documentclass[a4paper,10pt]{article}
\usepackage{myXsim}
\title{Nombre dérivé - Utilisation de la dérivée}
\tribe{1ST}
\date{Janvier 2020}
\pagestyle{empty}
%\geometry{left=15mm,right=15mm, bottom=8mm, top=5mm}
\begin{document}
\begin{exercise}[subtitle={Gestion hôtelière}]
Le nombre d'offre "séjour exclusif" vendues peut être modélisé par la fonction suivante $N(x) = -0.6x + 219$$x$ désigne le prix de vente en euro.
\begin{enumerate}
\item On se place dans le cas où le prix de vente est de 150\euro.
\begin{enumerate}
\item Combien d'offres seront vendues dans ce cas?
\item Quel sera alors les recettes pour cette vente?
\end{enumerate}
\item Mêmes questions dans le cas où le prix est de 200\euro? 300\euro.
\item Est-il vrai que plus le nombre d'offres vendues est élévé plus les recettes le seront aussi?
\item On veut étudier ces recettes. On note $R(x)$ la fonction qui modélise les recettes et où $x$ représente le prix de vente.
\begin{enumerate}
\item Expliquer que l'on a $R(x) = -0.6x^2 + 219x$
\item Calculer la dérivée de $R$.
\item Dresser le tableau de variations de $R$.
\item En déduire le prix de vente qui permet d'avoir une recette maximale. Combien vaut alors cette recette?
\end{enumerate}
\end{enumerate}
\end{exercise}
\begin{exercise}[subtitle={Coûts et recettes}]
Une entreprise fabrique des flacons de crème de beauté. Cette entreprise peut fabriquer jusqu'à 60 flacons par jour.
\begin{enumerate}
\item Chaque flacon est vendu 250\euro. On note $R(x)$ les recettes des ventes journalière des flacons où $x$ désigne le nombre de flacon produit. Déterminer l'expression de $R$ en fonction de $x$.
\item L'étude des coûts a mené à les modéliser par la fonction $C(x) = x^2 + 160x +800$. On note $B(x)$ la fonction qui modélise les bénéfices (recettes moins les coûts).
\begin{enumerate}
\item Est-il vrai que plus l'entreprise produit et vend plus elle fait des bénéfices?
\item Démontrer que $B(x) = -x^2 + 90x -800$
\item Calculer la dérivée $B'$ de $B$.
\item En déduire le tableau de variations de $B$
\item Combien de flacons doivent être produit pour maximiser les bénéfices? Quels seront alors ces bénéfices?
\end{enumerate}
\end{enumerate}
\end{exercise}
\printexercise{exercise}{1}
\printexercise{exercise}{2}
\end{document}